Đến nội dung

yellow nội dung

Có 365 mục bởi yellow (Tìm giới hạn từ 07-06-2020)



Sắp theo                Sắp xếp  

#376504 [CASIO]Chứng minh bán kính $r_k=\frac{1}{2\sqrt...

Đã gửi bởi yellow on 10-12-2012 - 12:01 trong Hình học

Cho dãy số ${b_n}$ được xác định như sau: $b_{n+2}=4b_{n+1}-b_n, b_1=4, b_2=14$
a) Chứng minh rằng diện tích tam giác với các cạnh $b_k-1,b_k,b_k+1$ là những số nguyên
b) Chứng minh rằng bán kính đường tròn nội tiếp tam giác được tính theo công thức
$$r_k=\frac{1}{2\sqrt{3}}[(2+\sqrt{3})^k-(2-\sqrt{3})^k]$$



#375886 [Casio] Số $2^{11}-1$ là số nguyên tố hay hợp số

Đã gửi bởi yellow on 07-12-2012 - 21:54 trong Các dạng toán khác

Bài 1: Số $2^{11}-1$ là số nguyên tố hay hợp số.
Bài 2: Tìm chữ số thập phân thứ $15$ sau dấu phẩy của $\sqrt{2003}$
Bài 3: Tìm tất cả các ước của $-2005$
Bài 4: Viết số sau dưới dạng phân số tối giản: $3124,142248$
------------------------------------------
p/s: Mọi người đưa ra lời giải dùm mình với nha, kết quả không quan trọng!



#360060 $\frac{10Pn-1}{Pn+1} -4=\frac{2}{n+1}$

Đã gửi bởi yellow on 08-10-2012 - 17:04 trong Tổ hợp - Xác suất và thống kê - Số phức

và {A_{n}}^{2} - {A_{n}}^{1}=3
mấy anh giải giúp em 2 bài này với

Ý bạn là thế này ak???
Bài 1: $\frac{10Pn-1}{Pn+1} -4=\frac{2}{n+1}$
Bài 2: ${A_{n}}^{2} - {A_{n}}^{1}=3$



#366672 $\frac{a^3+5}{a^3(b+c)}+\frac{b^3+5...

Đã gửi bởi yellow on 02-11-2012 - 22:20 trong Bất đẳng thức và cực trị

Xét các số thực dương $a,b,c$ thoả mãn $abc=1$. Chứng minh rằng:
$\frac{a^3+5}{a^3(b+c)}+\frac{b^3+5}{b^3(c+a)}+ \frac{c^3+5}{c^3(a+b)}$ $\geq9$



#358238 $\frac{a^2}{ma+nb}+\frac{b^2}...

Đã gửi bởi yellow on 02-10-2012 - 11:45 trong Bất đẳng thức và cực trị

Ta có $\frac{a^2}{ma+nb}+\frac{b^2}{mb+nc}+\frac{c^2}{mc+na} \geq \dfrac{(a+b+c)^2}{(m+n)(a+b+c)}=\frac{1}{m+n}(a+b+c)$

Đẳng thức xảy ra không anh.



#358234 $\frac{a^2}{ma+nb}+\frac{b^2}...

Đã gửi bởi yellow on 02-10-2012 - 11:37 trong Bất đẳng thức và cực trị

Cho $a,b,c,m,n>0$. Chứng minh rằng:
$\frac{a^2}{ma+nb}+\frac{b^2}{mb+nc}+\frac{c^2}{mc+na}\geq \frac{1}{m+n}(a+b+c)$



#358031 $\frac{a}{b}+\frac{b}{c...

Đã gửi bởi yellow on 01-10-2012 - 11:34 trong Bất đẳng thức và cực trị

Cho $a,b,c>0$. Chứng minh rằng: $\frac{a}{b}+\frac{b}{c}+\frac{c}{a}\geq \frac{a+b}{b+c}+\frac{b+c}{a+b}+1$
--------------------------------
Bài toán đã có tại đây:
http://diendantoanho...cabbcfracbcab1/



#358025 $\frac{a}{\sqrt{a^2+8bc}}+\...

Đã gửi bởi yellow on 01-10-2012 - 10:57 trong Bất đẳng thức và cực trị

Chứng minh rằng: $\frac{a}{\sqrt{a^2+8bc}}+\frac{b}{\sqrt{b^2+8ca}}+\frac{c}{\sqrt{c^2+8ab}} \geq 1 , \forall a, b, c > 0$



#344009 $\frac{2008+\frac{2007}{2}+\frac...

Đã gửi bởi yellow on 06-08-2012 - 15:31 trong Đại số

$\frac{2008 + \frac{2007}{2} + \frac{2006}{3} + \frac{2005}{4} + ... + \frac{2}{2007} + \frac{1}{2008}}{\frac{1}{2} + \frac{1}{3} + \frac{1}{4} + \frac{1}{5} + ... + \frac{1}{2008} + \frac{1}{2009}}$



#359821 $\frac{1719}{3976}=\frac{1}...

Đã gửi bởi yellow on 07-10-2012 - 18:08 trong Phương trình, hệ phương trình và bất phương trình

Tìm $a;b$ biết $\frac{1719}{3976}=\frac{1}{2+\frac{1}{3+\frac{1}{5+\frac{1}{a+\frac{1}{b}}}}}$



#358059 $\frac{1}{a_{1}+n-1}+\frac{...

Đã gửi bởi yellow on 01-10-2012 - 16:45 trong Bất đẳng thức - Cực trị

Cho $a_{1},a_{2},...,a_{n}$ là các số thực dương thoả mãn $a_{1}a_{2}...a_{n}=1$. Chứng minh rằng:
$\frac{1}{a_{1}+n-1}+\frac{1}{a_{2}+n-1}+...+\frac{1}{a_{n}+n-1}\leq 1$



#357773 $\frac{1}{a^3(b+c)}+\frac{1}...

Đã gửi bởi yellow on 30-09-2012 - 13:06 trong Bất đẳng thức và cực trị

Cho $a, b, c$ là các số thực dương thoả mãn $a.b.c=1$. Chứng minh rằng:
$\frac{1}{a^3(b+c)}+\frac{1}{b^3(c+a)}+\frac{1}{c^3(a+b)}\geq \frac{3}{2}$



#381090 $\frac{1}{1-x}+\frac{1}{1+x...

Đã gửi bởi yellow on 28-12-2012 - 07:53 trong Đại số

Rút gọn: $\frac{1}{1-x}+\frac{1}{1+x}+\frac{1}{1+x^2}+\frac{1}{1+x^4}+\frac{1}{1+x^8}+\frac{1}{1+x^{16}}$



#359820 $\frac{-94}{36}=a+\frac{b}{...

Đã gửi bởi yellow on 07-10-2012 - 18:04 trong Phương trình, hệ phương trình và bất phương trình

Tìm $a\in \mathbb{Z}; a_1,a_2,a_3,b_1,b_2,b_3\in \mathbb{N}$ biết:
$\frac{-94}{36}=a+\frac{b}{a_1+\frac{b_1}{a_2+\frac{b_2}{a_3+\frac{b_3}{3}}}}$



#356760 $$a+b+c\geq \frac{1+a}{1+b}+\fra...

Đã gửi bởi yellow on 26-09-2012 - 15:54 trong Bất đẳng thức và cực trị

Bài toán 2.
Ch0 các số thực dương $a,b,c$ thoả $abc=1$.Chứng minh rằng:
$$a+b+c\geq \frac{1+a}{1+b}+\frac{1+b}{1+c}+\frac{1+c}{1+a}$$

Đặt $1 + a = x$ ; $1 + b = y$ ; $1 + c = z$
Do $a, b, c$ là các số thực dương nên $x, y, z$ dương
Bất đẳng thức đã cho tương đương với:
$x + y + z - 3 \geq \frac{x}{y} + \frac{y}{z} + \frac{z}{x}$
$<=> x + y + z \geq \frac{x^{2}z + y^{2}x + z^{2}y + 3xyz}{xyz}$
$<=> xyz(x + y + z) \geq x^{2}z + y^{2}x + z^{2}y + 3xyz$
$<=> xyz(x + y + z) - x^{2}z - y^{2}x - z^{2}y \geq 3xyz$ (1)
Ta sẽ chứng minh bất đẳng thức (1), bất đẳng thức (1) hiển nhiên đúng:
$xyz(x + y + z) - x^{2}z - y^{2}x - z^{2}y$
$<=> x^{2}z(y - 1) + y^{2}x(z - 1) + z^{2}y(x - 1)$
Áp dụng bất đẳng thức Cauchy ta có:
$x^{2}z(y - 1) + y^{2}x(z - 1) + z^{2}y(x - 1) \geq 3xyz\sqrt[3]{(x-1)(y-1)(z-1)} = 3xyz\sqrt[3]{abc} = 3xyz$
$=> Đ.P.C.M$